Đến nội dung

Hình ảnh

Đơn giản

- - - - -

  • Please log in to reply
Chủ đề này có 4 trả lời

#1
MyLoveIs4Ever

MyLoveIs4Ever

    Sĩ quan

  • Thành viên
  • 441 Bài viết
CMR nếu $ 2^n \equiv -1 ( mod 3^k ) $ thì $ n | 3^{k-1} $

#2
Bé con

Bé con

    Binh nhất

  • Thành viên
  • 44 Bài viết
Liều mạng làm thử,sai cấm cười:
Ta có:
2^{2. 3^{k-1} } đồng dư 1 (mod 3^{k})
2^{2n} đồng dư 1 (mod 3^{k})
Suy ra 2^{2 (3^{k-1},n) } đồng dư 1 (mod 3^{k})
Nếu n không là ước 3^{k-1} suy ra (n,3^{k-1})=1 suy ra 4 đồng dư 1 (mod 3^{k}) suy ra k=1 suy ra bài toán sai nếu n=3,k=1
Nếu k>1 thì từ 4 đồng dư 1 mod 3^{k} dẫn đến mâu thuẫn,chứng tỏ bài toán đúng nếu k>1
(Đừng xóa bài em,em đang học đánh Latex)

#3
H.Quân- ĐHV

H.Quân- ĐHV

    An-tôn Páp-lô-vích Sê-Khốp

  • Thành viên
  • 530 Bài viết

CMR nếu $ 2^n \equiv -1 ( mod 3^k ) $ thì $ n | 3^{k-1} $

ta có$ (2,3^k) = 1$ nên

$ 2^{\phi (3^k) } \equiv 1 ( mod 3^k) $ :D $ 4^{3^{k-1} } \equiv 1 ( mod 3^k)$
mà $4^{n} \equiv 1 ( mod 3^k)$ nên $(n , 3^{k-1} ) = h$ ($ h$ là cấp )

thế thi $h = 3^t$ ta cm $t= k-1 $thật vậy nếu$ t<k-1$

ta có $4^{3^t} - 1 = 3 ( 4^{ 3^t - 1 } + 4^{3^t - 2} + ...+ 4+ 1 ) \vdots 3^k $

mà $4^{3^t - i} $ với $i=1 ,2,...,3^t -1 $ lập thành 1 hệ thặng dư đầy đủ mod $3^{k-1}$ ( do $ t $ min )

khi đó $4^{ 3^t - 1 } + 4^{3^t - 2} + ...+ 4+ 1 \equiv 3^t(3^t+1) : 2 ( mod 3^{k-1} ) $ :Rightarrow$ t = k-1 $ đpcm

Bài viết đã được chỉnh sửa nội dung bởi H.Quân- ĐHV: 16-04-2008 - 22:08

I hope for the best

Chẳng có gì đáng giá bằng nụ cười và tình yêu thương của bạn bè

Trên bước đường thành công không có dấu chân của kẻ lười biếng

#4
T*genie*

T*genie*

    Đường xa nặng bóng ngựa lười...

  • Quản lý Toán Ứng dụng
  • 1161 Bài viết

(Đừng xóa bài em,em đang học đánh Latex)


Anh gõ lại cho em được chứ?

$2^{2. 3^{k-1} }$ đồng dư 1 (mod $3^{k}$)
$2^{2n}$ đồng dư 1 (mod $3^{k}$)
Suy ra $2^{2 (3^{k-1},n) }$ đồng dư 1 (mod $3^{k}$)
Nếu n không là ước $3^{k-1}$ suy ra (n,$3^{k-1}$)=1 suy ra 4 đồng dư 1 (mod $3^{k}$) suy ra k=1 suy ra bài toán sai nếu n=3,k=1
Nếu k>1 thì từ 4 đồng dư 1 mod $3^{k}$ dẫn đến mâu thuẫn,chứng tỏ bài toán đúng nếu k>1


em chỉ cần chèn tex khi đánh công thức là ok.

#5
MyLoveIs4Ever

MyLoveIs4Ever

    Sĩ quan

  • Thành viên
  • 441 Bài viết
Bài nì nhiều cách CM lém , tạm thời có 2 cách nữa ( ý tưởng vẫn là căn nguyên thủy nhưng đi theo 2 hướng)

Dùng định lí sau
Với $ p $ nguyên tố lẻ nếu $ r $ là căn nguyên thủy modulo $ p $ và $ r $ cũng là căn nguyên thủy modulo $ p^2 $ thì $ r $ cũng là căn nguyên thủy modulo $ p^k $ mọi $ k \geq 3 $

CM ko khó mấy

Áp dụng vào là ok

Hoặc xét riêng bài toán này thì Chỉ cần CM $ 2 $ là căn nguyên thủy modulo $ 3^n $ là đủ
hay dùng bổ đề sau:
$ 2^{2.3^{n-1}} \equiv 1+3^n (mod 3^{n+1}) $




0 người đang xem chủ đề

0 thành viên, 0 khách, 0 thành viên ẩn danh